2024 AMC 10A Problems/Problem 1

Revision as of 01:18, 8 August 2024 by Skibidiuwghs (talk | contribs)
(diff) ← Older revision | Latest revision (diff) | Newer revision → (diff)

(problem statement left as an exercise to the reader)

Solution

Proof left as an exercise to the reader $\textbf{(A)}$